LSAT and Law School Admissions Forum

Get expert LSAT preparation and law school admissions advice from PowerScore Test Preparation.

User avatar
 Dave Killoran
PowerScore Staff
  • PowerScore Staff
  • Posts: 5852
  • Joined: Mar 25, 2011
|
#44139
Complete Question Explanation
(The complete setup for this game can be found here: lsat/viewtopic.php?t=16592)

The correct answer choice is (E)

Answer choice (A) is incorrect because J cannot be promoted this year.

Answer choice (B) is incorrect because O must be promoted this year.

Answer choice (C) is incorrect because S cannot be promoted two ranks during one promotion period.

Answer choice (D) is incorrect because at least one assistant must be promoted to associate.

Answer choice (E) is the correct answer choice.
 kevin.hussain24
  • Posts: 18
  • Joined: Oct 04, 2019
|
#73086
Hi,

I would like to understand why E is correct? Becuase doesnt R not vote to promote Lor S?

Also I'm usually pretty good at games, but these last 2 games in prep 18 where a bit different. Do you think mapping games or these specific grouping type game might appear on following tests?

Thank you
Kevin H
User avatar
 KelseyWoods
PowerScore Staff
  • PowerScore Staff
  • Posts: 1079
  • Joined: Jun 26, 2013
|
#73142
Hi Kevin!

According to the scenario, an assistant is promoted to associate when a majority of the higher ranking staff vote for them. Both the partners H & R and the associate O rank higher than than the assistants. So to get promoted, an assistant just needs 2 out of those 3 votes. J can't get promoted in that first round because he can't get votes from O or H. But every other assistant has the potential to get the 2 out of 3 votes they need to be promoted. R doesn't vote for L or S, but both H and O could vote for L and S.

They can always throw a rarer game type into the mix on any given exam. Even if it's a familiar type, they can give you rules or restrictions that you haven't seen before that may be tricky. The best thing you can do is to practice as many examples of games as you can and have a strategy for how to approach games that seem unfamiliar to you (e.g., leave them for last, diagram what you can, go back to your diagram if you get to the questions and realize you've misunderstood a key rule, etc.).

Hope this helps!

Best,
Kelsey
 Mastering_LSAT
  • Posts: 35
  • Joined: Jul 30, 2020
|
#82740
Hello! I hope you could clarify the following. This is what I am confused about.

We know that "an assistant is promoted to associate when a majority of higher-ranking staff votes for promotion". We also know that "everyone eligible votes on every promotion", i.e. there are no abstentions and everyone votes either in favor or against, not both. It is also clear from the rules that "votes for promotion" of someone are sufficient for anyone's promotion to the next level. Based on the rules and analyzing the question stem, we do know that W will be promoted to an associate after this year's review, it's cannot be true that he would remain an assistant. I do not understand where we can find even 1% of justification (I'm referring to the truth scale from the PowerScore Bible) that W could remain an assistant after this year's review? I understand why (A)-(D) are all wrong answers, but I can't agree with (E) either. Please explain how on Earth it is possible for W to remain an assistant when based on the rules we know that there is 0% chance that he would remain one (it is cannot be true) and that he will be guaranteed (100%) promoted to an associate.

Many thanks!
 Adam Tyson
PowerScore Staff
  • PowerScore Staff
  • Posts: 5153
  • Joined: Apr 14, 2011
|
#83384
I think you might be making an unwarranted assumption about who votes for whom, Mastering_LSAT. We know that H never votes for W, but that does not mean that everyone else DOES vote for W. It is entirely possible that NOBODY votes in favor of W getting promoted and that all the people above W vote against her. The rules only tell us who does not vote for promoting someone, and you cannot use those rules to infer anything about who does for those promotions. Thus, another perfectly acceptable solution would include W being the ONLY Assistant promoted to Associate.

Put another way, if I told you that both O and R voted against promoting W, could you tell me which rule I broke?

Be careful about those assumptions!

Get the most out of your LSAT Prep Plus subscription.

Analyze and track your performance with our Testing and Analytics Package.